Saltar al contenido principal
LibreTexts Español

5.8: La Matriz de una Transformación Lineal II

  • Page ID
    114503
  • \( \newcommand{\vecs}[1]{\overset { \scriptstyle \rightharpoonup} {\mathbf{#1}} } \) \( \newcommand{\vecd}[1]{\overset{-\!-\!\rightharpoonup}{\vphantom{a}\smash {#1}}} \)\(\newcommand{\id}{\mathrm{id}}\) \( \newcommand{\Span}{\mathrm{span}}\) \( \newcommand{\kernel}{\mathrm{null}\,}\) \( \newcommand{\range}{\mathrm{range}\,}\) \( \newcommand{\RealPart}{\mathrm{Re}}\) \( \newcommand{\ImaginaryPart}{\mathrm{Im}}\) \( \newcommand{\Argument}{\mathrm{Arg}}\) \( \newcommand{\norm}[1]{\| #1 \|}\) \( \newcommand{\inner}[2]{\langle #1, #2 \rangle}\) \( \newcommand{\Span}{\mathrm{span}}\) \(\newcommand{\id}{\mathrm{id}}\) \( \newcommand{\Span}{\mathrm{span}}\) \( \newcommand{\kernel}{\mathrm{null}\,}\) \( \newcommand{\range}{\mathrm{range}\,}\) \( \newcommand{\RealPart}{\mathrm{Re}}\) \( \newcommand{\ImaginaryPart}{\mathrm{Im}}\) \( \newcommand{\Argument}{\mathrm{Arg}}\) \( \newcommand{\norm}[1]{\| #1 \|}\) \( \newcommand{\inner}[2]{\langle #1, #2 \rangle}\) \( \newcommand{\Span}{\mathrm{span}}\)\(\newcommand{\AA}{\unicode[.8,0]{x212B}}\)

    Resultados

    1. Encontrar la matriz de una transformación lineal con respecto a bases generales.

    Comenzamos esta sección con un lema importante.

    Lema\(\PageIndex{1}\): Mapping of a Basis

    \(T: \mathbb{R}^n \mapsto \mathbb{R}^n\)Déjese ser un isomorfismo. Entonces\(T\) mapea cualquier base de\(\mathbb{R}^n\) a otra base para\(\mathbb{R}^n\).

    Por el contrario, si\(T: \mathbb{R}^n \mapsto \mathbb{R}^n\) es una transformación lineal que mapea una base de\(\mathbb{R}^n\) a otra base de\(\mathbb{R}^n\), entonces es un isomorfismo.

    Prueba

    Primero, supongamos que\(T:\mathbb{R}^n \mapsto \mathbb{R}^n\) es una transformación lineal que es uno a uno y sobre. Dejemos\(\left\{ \vec{v}_{1},\cdots ,\vec{v}_{n}\right\}\) ser una base para\(\mathbb{R}^n\). Deseamos demostrar que también\(\left\{ T(\vec{v}_{1}),\cdots , T(\vec{v}_{n})\right\}\) es una base para\(\mathbb{R}^n\).

    Primero considere por qué es linealmente independiente. Supongamos\(\sum_{k=1}^{n}a_{k}T(\vec{v}_{k})=\vec{0}\). Entonces por linealidad tenemos\(T\left( \sum_{k=1}^{n}a_{k}\vec{v}_{k}\right) =\vec{0}\) y ya que\(T\) es uno a uno, se deduce de eso\(\sum_{k=1}^{n}a_{k}\vec{v}_{k}=\vec{0}\). Esto requiere que cada uno\(a_{k}=0\) porque\(\left\{ \vec{v}_{1},\cdots, \vec{v}_{n}\right\}\) sea independiente, y de ello se deduce que\(\left\{ T(\vec{v}_{1}),\cdots , T(\vec{v}_{n})\right\}\) es linealmente independiente.

    Siguiente toma\(\vec{w}\in \mathbb{R}^n.\) Ya que\(T\) está en, existe\(\vec{v}\in \mathbb{R}^n\) tal que\(T(\vec{v})=\vec{w}\). Ya que\(\left\{ \vec{v}_{1},\cdots ,\vec{v}_{n}\right\}\) es una base, en particular es un conjunto que abarca y hay escalares\(b_{k}\) tales que\(T\left( \sum_{k=1}^{n}b_{k}\vec{v} _{k}\right) =T\left( \vec{v}\right) =\vec{w}\). Por lo tanto\(\vec{w} =\sum_{k=1}^{n}b_{k}T(\vec{v}_{k})\) que se encuentra en el\(\mathrm{span}\left\{ T(\vec{v}_{1}),\cdots , T(\vec{v}_{n})\right\} .\) Por lo tanto,\(\left\{ T(\vec{v}_{1}),\cdots ,T(\vec{v}_{n}) \right\}\) es una base como se reivindica.

    Supongamos ahora que\(T: \mathbb{R}^n \mapsto \mathbb{R}^n\) es una transformación lineal tal que\(T(\vec{v}_{i})=\vec{w}_{i}\) donde\(\left\{\vec{v} _{1},\cdots ,\vec{v}_{n}\right\}\) y\(\left\{ \vec{w}_{1},\cdots , \vec{w}_{n}\right\}\) son dos bases para\(\mathbb{R}^n\).

    Para demostrar que\(T\) es uno a uno, vamos\(T\left( \sum_{k=1}^{n}c_{k}\vec{v}_{k}\right) =\vec{0}\). Entonces\(\sum_{k=1}^{n}c_{k}T(\vec{v}_{k})=\sum_{k=1}^{n}c_{k}\vec{w}_{k}=\vec{ 0}\). De ello se deduce que cada uno\(c_{k} = 0\) porque se le da que\(\left\{ \vec{w} _{1},\cdots ,\vec{w}_{n}\right\}\) es linealmente independiente. De ahí\(T\left( \sum_{k=1}^{n}c_{k}\vec{v}_{k}\right) =\vec{0}\) implica eso\(\sum_{k=1}^{n}c_{k}\vec{v}_{k}=\vec{0}\) y así lo\(T\) es uno a uno.

    Para mostrar que\(T\) está en, dejar\(\vec{w}\) ser un vector arbitrario en\(\mathbb{R}^n\). Este vector se puede escribir como\(\vec{w} = \sum_{k=1}^{n}d_k\vec{w}_k = \sum_{k=1}^{n}d_{k}T(\vec{v}_{k})=T\left( \sum_{k=1}^{n}d_{k} \vec{v}_{k}\right) .\) Por lo tanto, también\(T\) está en.

    Consideremos ahora una definición importante.

    Definición\(\PageIndex{1}\): Coordinate Vector

    Dejar\(B = \left\{ \vec{v}_1, \vec{v}_2, \cdots, \vec{v}_n \right\}\) ser una base para\(\mathbb{R}^n\) y dejar\(\vec{x}\) ser un vector arbitrario en\(\mathbb{R}^n\). Entonces\(\vec{x}\) se representa de manera única como\(\vec{x} = a_1\vec{v}_1 + a_2\vec{v}_2 + \cdots + a_n\vec{v}_n\) para los escalares\(a_1, \cdots, a_n\).

    El vector de coordenadas de\(\vec{x}\) con respecto a la base\(B\), escrito\(C_B(\vec{x})\) o\([\vec{x}]_B\), viene dado por\[C_B(\vec{x}) = C_B \left( a_1\vec{v}_1 + a_2\vec{v}_2 + \cdots + a_n\vec{v}_n \right) = \left [ \begin{array}{c} a_1 \\ a_2 \\ \vdots \\ a_n \end{array} \right ]\nonumber \]

    Considera el siguiente ejemplo.

    Ejemplo\(\PageIndex{1}\): Coordinate Vector

    Dejar\(B = \left\{ \left [ \begin{array}{r} 1 \\ 0 \end{array} \right ], \left [ \begin{array}{r} -1 \\ 1 \end{array} \right ] \right\}\) ser una base de\(\mathbb{R}^2\) y dejar\(\vec{x} = \left [ \begin{array}{r} 3 \\ -1 \end{array} \right ]\) ser un vector en\(\mathbb{R}^2\). Encontrar\(C_B(\vec{x})\).

    Solución

    Primero, tenga en cuenta que el orden de la base es importante así etiquetar los vectores en la base\(B\) como\[B = \left\{ \left [ \begin{array}{r} 1 \\ 0 \end{array} \right ], \left [ \begin{array}{r} -1 \\ 1 \end{array} \right ] \right\} = \left\{ \vec{v}_1, \vec{v}_2 \right\}\nonumber \] Ahora necesitamos encontrar\(a_1, a_2\) tal que\(\vec{x} = a_1 \vec{v}_1 + a_2 \vec{v}_2\), es decir:\[\left [ \begin{array}{r} 3 \\ -1 \end{array} \right ] = a_1 \left [ \begin{array}{r} 1 \\ 0 \end{array} \right ] + a_2 \left [ \begin{array}{r} -1 \\ 1 \end{array} \right ]\nonumber \] Resolver este sistema da\(a_1 = 2, a_2 = -1\). Por lo tanto, el vector de coordenadas de\(\vec{x}\) con respecto a la base\(B\) es\[C_B(\vec{x}) = \left [ \begin{array}{r} a_1 \\ a_2 \end{array}\right ] = \left [ \begin{array}{r} 2 \\ -1 \end{array} \right ]\nonumber \]

    Dada cualquier base\(B\), se puede verificar fácilmente que la función de coordenadas es en realidad un isomorfismo.

    Teorema\(\PageIndex{1}\): \(C_B\)Transformation is a Linear

    Para cualquier base\(B\) de\(\mathbb{R}^n\), la función de coordenadas\[C_B: \mathbb{R}^n \rightarrow \mathbb{R}^n\nonumber \] es una transformación lineal, y además un isomorfismo.

    Ahora discutimos el resultado principal de esta sección, es decir, cómo representar una transformación lineal con respecto a diferentes bases.

    Teorema\(\PageIndex{2}\): The Matrix of a Linear

    Dejar\(T: \mathbb{R}^n \mapsto \mathbb{R}^m\) ser una transformación lineal, y dejar\(B_1\) y\(B_2\) ser bases de\(\mathbb{R}^{n}\) y\(\mathbb{R}^{m}\) respectivamente.

    Entonces lo siguiente sostiene\[C_{B_2} T = M_{B_{2} B_{1}} C_{B_1} \label{matrixequation}\] donde\(M_{B_{2} B_{1}}\) es una\(m \times n\) matriz única.

    Si la base\(B_1\) es dada por\(B_1=\{ \vec{v}_1, \cdots, \vec{v}_n \}\) en este orden, entonces\[M_{B_{2} B_{1}} = \left [ C_{B_2}(T(\vec{v}_1)) \; C_{B_2}(T(\vec{v}_2)) \; \cdots C_{B_2}(T(\vec{v}_n)) \right ]\nonumber \]

    Prueba

    La ecuación anterior\(\eqref{matrixequation}\) puede ser representada por el siguiente diagrama. \[\begin{array}{rrcll} & & T & & \\ & \mathbb{R}^n & \rightarrow & \mathbb{R}^m & \\ & C_{B_{1} }\downarrow & \circ & \downarrow C_{B_{2} } & \\ & \mathbb{R}^{n} & \rightarrow & \mathbb{R}^{m} & \\ & & M_{B_{2} B_{1} } & & \end{array}\nonumber \]

    Ya que\(C_{B_1}\) es un isomorfismo, entonces la matriz que estamos buscando es la matriz de la transformación lineal\[C_{B_2} T C^{-1}_{B_1} : \mathbb{R}^n \mapsto \mathbb{R}^m.\nonumber \] Por Teorema 5.2.2, las columnas están dadas por la imagen de la base estándar\(\left\{ \vec{e}_1, \vec{e}_2, \cdots, \vec{e}_n \right\}\). Pero ya que\(C^{-1}_{B_1}( \vec{e}_i) = \vec{v}_i\), fácilmente obtenemos eso\[\begin{array}{ll} M_{B_{2} B_{1}} & = \left [ C_{B_2}T C^{-1}_{B_1} (\vec{e}_1) \;\; C_{B_2}T C^{-1}_{B_1} (\vec{2}_2) \;\; \cdots \;\; C_{B_2}T C^{-1}_{B_1} (\vec{e}_n) \right ] \\ & = \left [ C_{B_2}(T(\vec{v}_1)) \;\; C_{B_2}(T(\vec{v}_2)) \;\; \cdots \;\; C_{B_2}(T(\vec{v}_n)) \right ] \end{array}\nonumber \] y esto completa la prueba.

    Considera el siguiente ejemplo.

    Ejemplo\(\PageIndex{2}\): Matrix of a Linear

    Dejar\(T: \mathbb{R}^2 \mapsto \mathbb{R}^2\) ser una transformación lineal definida por\(T \left( \left [ \begin{array}{r} a \\ b \end{array} \right ] \right) = \left [ \begin{array}{r} b \\ a \end{array} \right ]\).

    Considere las dos bases\[B_1 = \left\{ \vec{v}_{1}, \vec{v}_{2} \right\} = \left\{ \left [ \begin{array}{r} 1 \\ 0 \end{array}\right ], \left [ \begin{array}{r} -1 \\ 1 \end{array} \right ] \right\}\nonumber \] y\[B_2 = \left\{ \left [ \begin{array}{r} 1 \\ 1 \end{array} \right ], \left [ \begin{array}{r} 1 \\ -1 \end{array} \right ] \right\}\nonumber \]

    Encuentra la matriz\(M_{B_2,B_1}\) de\(T\) con respecto a las bases\(B_1\) y\(B_2\).

    Solución

    Por teorema\(\PageIndex{2}\), las columnas de\(M_{B_{2} B_{1}}\) son los vectores de coordenadas de\(T(\vec{v}_{1}), T(\vec{v}_{2})\) con respecto a\(B_2\).

    Dado que\[T \left( \left [ \begin{array}{r} 1 \\ 0 \end{array}\right ] \right) = \left [ \begin{array}{r} 0 \\ 1 \end{array}\right ] ,\nonumber \] un cálculo estándar arroja\[\left [ \begin{array}{r} 0 \\ 1 \end{array}\right ] = \left(\frac{1}{2} \right)\left [ \begin{array}{r} 1 \\ 1 \end{array} \right ] + \left(-\frac{1}{2} \right) \left [ \begin{array}{r} 1 \\ -1 \end{array} \right ],\nonumber \] la primera columna de\(M_{B_{2} B_{1}}\) is\(\left [ \begin{array}{r} \frac{1}{2}\\ -\frac{1}{2} \end{array}\right ]\).

    La segunda columna se encuentra de manera similar. Tenemos\[T \left( \left [ \begin{array}{r} -1 \\ 1 \end{array} \right ] \right) = \left [ \begin{array}{r} 1 \\ -1 \end{array}\right ] ,\nonumber \] y con respecto a\(B_2\) calcular:\[\left [ \begin{array}{r} 1 \\ -1 \end{array}\right ] = 0 \left [ \begin{array}{r} 1 \\ 1 \end{array} \right ] + 1 \left [ \begin{array}{r} 1 \\ -1 \end{array} \right ]\nonumber \] De ahí la segunda columna de\(M_{B_{2} B_{1}}\) viene dada por\(\left [ \begin{array}{r} 0 \\ 1 \end{array} \right ]\). Así obtenemos\[M_{B_{2} B_{1}} = \left [ \begin{array}{rr} \frac{1}{2} & 0 \\ -\frac{1}{2} & 1 \end{array} \right ]\nonumber \]

    Podemos verificar que esta es la matriz correcta\(M_{B_{2} B_{1}}\) en el ejemplo específico\[\vec{v} = \left [ \begin{array}{r} 3 \\ -1 \end{array} \right ]\nonumber \] Primero aplicando\(T\) da\[T( \vec{v} ) = T \left( \left [ \begin{array}{r} 3 \\ -1 \end{array} \right ] \right) = \left [ \begin{array}{r} -1\\ 3 \end{array} \right ]\nonumber \] y uno puede computar eso\[C_{B_2} \left( \left [ \begin{array}{r} -1 \\ 3 \end{array} \right ] \right) = \left [ \begin{array}{r} 1\\ -2 \end{array} \right ] .\nonumber \]

    Por otro lado, uno computa\(C_{B_1}( \vec{v})\) como\[C_{B_1} \left( \left [ \begin{array}{r} 3 \\ -1 \end{array} \right ] \right) = \left [ \begin{array}{r} 2\\ -1 \end{array} \right ] ,\nonumber \] y finalmente aplicando\(M_{B_1 B_2}\) da\[\left [ \begin{array}{rr} \frac{1}{2} & 0 \\ -\frac{1}{2} & 1 \end{array} \right ] \left [ \begin{array}{r} 2 \\ -1 \end{array}\right ] = \left [ \begin{array}{r} 1 \\ -2 \end{array} \right ]\nonumber \] como arriba.

    Vemos que el mismo vector resulta de cualquiera de los dos métodos, como sugiere el Teorema\(\PageIndex{2}\).

    Si las bases\(B_1\) y\(B_2\) son iguales, digamos\(B\), entonces escribimos\(M_{B}\) en lugar de\(M_{B B}\). El siguiente ejemplo ilustra cómo calcular dicha matriz. Tenga en cuenta que esto es lo que hicimos antes cuando consideramos\(B_1=B_2\) que sólo era la base estándar.

    Ejemplo\(\PageIndex{3}\): Matrix of a Linear Transformation with respect to an Arbitrary

    Considerar la base\(B\) de\(\mathbb{R}^3\) dada por\[B = \{\vec{v}_1 , \vec{v}_2, \vec{v}_3 \} = \left\{ \left [ \begin{array}{r} 1 \\ 0 \\ 1 \end{array} \right ] ,\left [ \begin{array}{r} 1 \\ 1 \\ 1 \end{array} \right ] ,\left [ \begin{array}{r} -1 \\ 1 \\ 0 \end{array} \right ] \right\}\nonumber \] Y dejar que\(T :\mathbb{R}^{3}\mapsto \mathbb{R}^{3}\) sea la transformación lineal definida\(B\) como:\[T\left [ \begin{array}{r} 1 \\ 0 \\ 1 \end{array} \right ] =\left [ \begin{array}{r} 1 \\ -1 \\ 1 \end{array} \right ] ,T \left [ \begin{array}{c} 1 \\ 1 \\ 1 \end{array} \right ] =\left [ \begin{array}{r} 1 \\ 2 \\ -1 \end{array} \right ] ,T\left [ \begin{array}{r} -1 \\ 1 \\ 0 \end{array} \right ] =\left [ \begin{array}{r} 0 \\ 1 \\ 1 \end{array} \right ]\nonumber \]

    1. Encuentra la matriz\(M_{B}\) de\(T\) relativo a la base\(B\).
    2. Entonces encuentra la matriz habitual de\(T\) con respecto a la base estándar de\(\mathbb{R}^{3}\).
    Solución

    Ecuación\(\eqref{matrixequation}\) da\(C_BT=M_{B}C_B\), y así\(M_{B} = C_BTC^{-1}_B\).

    Ahora\(C_B(\vec{v}_i) = \vec{e}_i\), entonces la matriz de\(C_B^{-1}\) (con respecto a la base estándar) viene dada por\[\left [ C_B^{-1}(\vec{e}_1) \;\; C_B^{-1}(\vec{e}_2) \;\; C_B^{-1}(\vec{e}_2) \right ] = \left [ \begin{array}{rrr} 1 & 1 & -1 \\ 0 & 1 & 1 \\ 1 & 1 & 0 \end{array} \right ]\nonumber \] Además la matriz de\(T C_B^{-1}\) es dada por\[\left [ TC_B^{-1}(\vec{e}_1) \;\; TC_B^{-1}(\vec{e}_2) \;\; TC_B^{-1}(\vec{e}_2) \right ] = \left [ \begin{array}{rrr} 1 & 1 & 0 \\ -1 & 2 & 1 \\ 1 & -1 & 1 \end{array} \right ]\nonumber \] Así\[\begin{array}{ll} M_{B} & = C_BTC^{-1}_B = [C^{-1}_B]^{-1} [TC^{-1}_B] \\ & = \left [ \begin{array}{rrr} 1 & 1 & -1 \\ 0 & 1 & 1 \\ 1 & 1 & 0 \end{array} \right ] ^{-1}\left [ \begin{array}{rrr} 1 & 1 & 0 \\ -1 & 2 & 1 \\ 1 & -1 & 1 \end{array} \right ] \\ &=\left [ \begin{array}{rrr} 2 & -5 & 1 \\ -1 & 4 & 0 \\ 0 & -2 & 1 \end{array} \right ] \end{array}\nonumber \]

    Considera cómo funciona esto. Dejar\(\vec{b} = \left [ \begin{array}{r} b_1 \\ b_2 \\ b_3 \end{array} \right ]\) ser un vector arbitrario en\(\mathbb{R}^3\).

    Aplicar\(C^{-1}_{B}\)\(\vec{b}\) a para obtener\[b_1\left [ \begin{array}{r} 1 \\ 0 \\ 1 \end{array} \right ] + b_2\left [ \begin{array}{r} 1 \\ 1 \\ 1 \end{array} \right ] + b_3\left [ \begin{array}{r} -1 \\ 1 \\ 0 \end{array} \right ]\nonumber \] Aplicar\(T\) a esta combinación lineal para obtener\[b_1\left [ \begin{array}{r} 1 \\ -1 \\ 1 \end{array} \right ] + b_2\left [ \begin{array}{r} 1 \\ 2 \\ -1 \end{array} \right ] + b_3\left [ \begin{array}{r} 0 \\ 1 \\ 1 \end{array} \right ] =\left [ \begin{array}{c} b_1+b_2 \\ -b_1 + 2b_2+ b_3 \\ b_1-b_2+b_3 \end{array} \right ]\nonumber \] Ahora toma la matriz\(M_{B}\) de la transformación (como se encontró anteriormente) y multiplícala por\(\vec{b}\). \[\left [ \begin{array}{rrr} 2 & -5 & 1 \\ -1 & 4 & 0 \\ 0 & -2 & 1 \end{array} \right ] \left [ \begin{array}{c} b_1 \\ b_2 \\ b_3 \end{array} \right ] =\left [ \begin{array}{c} 2b_1-5b_2+b_3 \\ -b_1 + 4b_2 \\ -2b_2 + b_3 \end{array} \right ]\nonumber \]¿Es este el vector de coordenadas de lo anterior relativo a la base dada? Comprobamos de la siguiente manera. \[\left( 2b_1-5b_2+b_3\right) \left [ \begin{array}{c} 1 \\ 0 \\ 1 \end{array} \right ] +\left( -b_1 + 4b_2\right) \left [ \begin{array}{c} 1 \\ 1 \\ 1 \end{array} \right ] +\left( -2b_2+b_3\right) \left [ \begin{array}{c} -1 \\ 1 \\ 0 \end{array} \right ]\nonumber \]\[= \left [ \begin{array}{c} b_1+b_2 \\ -b_1 + 2b_2+b_3 \\ b_1-b_2+b_3 \end{array} \right ]\nonumber \]Ya ves que es lo mismo.

    Ahora vamos a encontrar la matriz de\(T\) con respecto a la base estándar. Déjese\(A\) ser esta matriz. Es decir, multiplicar por\(A\) es lo mismo que hacer\(T\). Así\[A\left [ \begin{array}{rrr} 1 & 1 & -1 \\ 0 & 1 & 1 \\ 1 & 1 & 0 \end{array} \right ] =\left [ \begin{array}{rrr} 1 & 1 & 0 \\ -1 & 2 & 1 \\ 1 & -1 & 1 \end{array} \right ]\nonumber \] De ahí Por\[A=\left [ \begin{array}{rrr} 1 & 1 & 0 \\ -1 & 2 & 1 \\ 1 & -1 & 1 \end{array} \right ] \left [ \begin{array}{rrr} 1 & 1 & -1 \\ 0 & 1 & 1 \\ 1 & 1 & 0 \end{array} \right ] ^{-1}=\left [ \begin{array}{rrr} 0 & 0 & 1 \\ 2 & 3 & -3 \\ -3 & -2 & 4 \end{array} \right ]\nonumber \] supuesto esta es una matriz muy diferente a la matriz de la transformación lineal con respecto a la base no estándar.


    This page titled 5.8: La Matriz de una Transformación Lineal II is shared under a CC BY 4.0 license and was authored, remixed, and/or curated by Ken Kuttler (Lyryx) via source content that was edited to the style and standards of the LibreTexts platform; a detailed edit history is available upon request.